if the base of a right angled triangle is 4cm and its area is 20^2, find its height

Answers

Answer 1

Answer:

80cm

Step-by-step explanation:

i worked it out like this

2x4=8

then that means

20x4=80

Answer 2

Answer:

80 is the answer


Related Questions

a discount voucher offering 15% off is used to pay a bill. after using the voucher the bill is reduced to £36.72.
how much was the bill before applying the voucher discount?

Answers

36.72 is 85%
hence 100% = (36.72/85) × 100

Kyla makes a triangular school pennant. The area of the triangle is 180 square inches. The base of the pennant is z inches long. The height is 6 inches longer than twice the base length.


What is the height of the pennant? Recall the formula


A = bh.

Answers

Answer:

Height of the pennant is 30 inches.

Step-by-step explanation:

Given that:

Area of pennant = 180 sq inches

Base of pennant = z inches

Height of pennant = (2z + 6) inches

Also, it is a triangular pennant and area of a triangle can be given as:

[tex]A = \dfrac{1}{2} \times Base\times Height[/tex]

Putting the values in above formula:

[tex]180 = \dfrac{1}{2} \times z \times (2z+6)\\\Rightarrow 360 = 2z^{2} + 6z\\\Rightarrow 180 = z^{2} + 3z\\\Rightarrow z^{2} + 3z -180 = 0\\\Rightarrow z^{2} + 15z -12z -180 = 0\\\Rightarrow z(z + 15) -12(z+15) = 0\\\Rightarrow (z + 15) (z-12) = 0\\\Rightarrow z = 12\ or\ z=-15[/tex]

Value of z can not be negative, so value of Base, z = 12 inches.

Height is given as 2z + 6 so, height = 2[tex]\times[/tex]12 +6 = 30 inches

Answer:

C.30 inches

Step-by-step explanation:

The weight of oranges growing in an orchard is normally distributed with a mean weight of 6 oz. and a standard deviation of 0.5 oz. Using the empirical rule, determine what interval would represent weights of the middle 95% of all oranges from this orchard.

Answers

Answer:

The interval that would represent weights of the middle 95% of all oranges from this orchard is from 5 oz to 7 oz.

Step-by-step explanation:

The Empirical Rule states that, for a normally distributed random variable:

68% of the measures are within 1 standard deviation of the mean.

95% of the measures are within 2 standard deviation of the mean.

99.7% of the measures are within 3 standard deviations of the mean.

In this problem, we have that:

Mean = 6

Standard deviation = 0.5

Middle 95% of weights:

By the Empirical Rule, within 2 standard deviations of the mean.

6 - 2*0.5 = 5

6 + 2*0.5 = 7

The interval that would represent weights of the middle 95% of all oranges from this orchard is from 5 oz to 7 oz.

The interval representing the weights of the middle 95% of all oranges from this orchard is from 5 oz to 7 oz.

The Empirical Rule states that for a normally distributed random variable:

68% of the measures are within 1 standard deviation of the mean.

What is the empirical rule?

The empirical rule says that, in a standard data set, virtually every piece of data will fall within three standard deviations of the mean.

95% of the measures are within 2 standard deviations of the mean.

99.7% of the measures are within 3 standard deviations of the mean.

In this problem, we have that:

Mean = 6

Standard deviation = 0.5

Middle 95% of weights

By the Empirical Rule, within 2 standard deviations of the mean.

6 - 2*0.5 = 5

6 + 2*0.5 = 7

The interval representing the weights of the middle 95% of all oranges from this orchard is from 5 oz to 7 oz.

To learn more about the empirical rule visit:

https://brainly.com/question/24244232

#SPJ5

If A = (0, 0) and B = (6, 3) what is the length of overline AB ?

Answers

Answer:

A= (0,0) and B = (6,3)

We can find the length AB with the following formula:

[tex] d = \sqrt{(x_B -x_A)^2 +(y_B -y_A)^2}[/tex]

And replacing we got:

[tex] d = \sqrt{(6-0)^2 +(3-0)^2} = \sqrt{45}= 3\sqrt{5}[/tex]

So then the length AB would be [tex] 3\sqrt{5}[/tex]

Step-by-step explanation:

For this case we have the following two points:

A= (0,0) and B = (6,3)

We can find the length AB with the following formula:

[tex] d = \sqrt{(x_B -x_A)^2 +(y_B -y_A)^2}[/tex]

And replacing we got:

[tex] d = \sqrt{(6-0)^2 +(3-0)^2} = \sqrt{45}= 3\sqrt{5}[/tex]

So then the length AB would be [tex] 3\sqrt{5}[/tex]

Answer:6.71

Step-by-step explanation: awesomeness

Determine the maximized area of a rectangle that has a perimeter equal to 56m by creating and solving a quadratic equation. What is the length and width?

Answers

Answer:

Area of rectangle = [tex]196\,m^2[/tex]

Length of rectangle = 14 m

Width of rectangle = 14 m

Step-by-step explanation:

Given:

Perimeter of rectangle is 56 m

To find: the maximized area of a rectangle and the length and width

Solution:

A function [tex]y=f(x)[/tex] has a point of maxima at [tex]x=x_0[/tex] if [tex]f''(x_0)<0[/tex]

Let x, y denotes length and width of the rectangle.

Perimeter of rectangle = 2( length + width )

[tex]=2(x+y)[/tex]

Also, perimeter of rectangle is equal to 56 m.

So,

[tex]56=2(x+y)\\x+y=28\\y=28-x[/tex]

Let A denotes area of rectangle.

A = length × width

[tex]A=xy\\=x(28-x)\\=28x-x^2[/tex]

Differentiate with respect to x

[tex]\frac{dA}{dx}=28-2x[/tex]

Put [tex]\frac{dA}{dx}=0[/tex]

[tex]28-2x=0\\2x=28\\x=14[/tex]

Also,

[tex]\frac{d^2A}{dx^2}=-2<0[/tex]

At x = 14, [tex]\frac{d^2A}{dx^2}=-2<0[/tex]

So, x = 14 is a point of maxima

So,

[tex]y=28-x=28-14=14[/tex]

Area of rectangle:

[tex]A=xy=14(14)=196\,m^2[/tex]

Length of rectangle = 14 m

Width of rectangle = 14 m

Solve the system of equations by finding the reduced row-echelon form of the augmented matrix for the system of equations.
2x + y + 4z = 16
5x - 2y + 2z = -1
X + 2y - 32 = -9
a. (-1, 2, 22)
c. (-1, 2, 4)
b. (-10, 22, 42)
d. (-10, 2, 22)

Answers

The correct answer will be D

The solution to the system of equations is (-1, 2, 4), which is equal to choice (c) (-1, 2, 4).

What is the system of equations?

One or many equations having the same number of unknowns that can be solved simultaneously are called simultaneous equations. And the simultaneous equation is the system of equations.

To solve the system of equations, we can represent it in an augmented matrix and use row operations to find its reduced row-echelon form.

The augmented matrix for the system is:

[ 2  1  4 | 16 ]

[ 5 -2  2 | -1 ]

[ 1  2 -3 | -9 ]

We want to get the matrix in reduced row-echelon form.

We can start by using row operations to get a 1 in the upper left corner:

R(1/2) -> R1:

[ 1 1/2  2 |  8 ]

[ 5 -2  2 | -1 ]

[ 1  2 -3 | -9 ]

Now we want to get zeros in the first column below the pivot element (1). We can do this by subtracting 5 times the first row from the second row, and subtracting the first row from the third row:

-5R1 + R2 -> R2:

-R1 + R3 -> R3:

[ 1  1/2   2  |  8 ]

[ 0 -9/2  -8  | -41 ]

[ 0  3/2 -5  | -17 ]

R2 + R3 -> R3:

[ 1  1/2   2  |  8 ]

[ 0 -9/2  -8  | -41 ]

[ 0  -3    -13 | -58 ]

We can continue with row operations to get a 1 in the second row, second column:

(-2/9)R2 -> R2:

[ 1  1/2   2    |  8 ]

[ 0   1   16/9  |  82/9 ]

[ 0  -3    -13 | -58 ]

3R2 + R3 -> R3:

[ 1  1/2   2    |  8 ]

[ 0   1   16/9  |  82/9 ]

[ 0  0    -23/3 | -92/3 ]

Finally, we can get a 1 in the third row, third column:

(-3/23)R3 -> R3:

[ 1  1/2   2    |  8 ]

[ 0   1   16/9  |  82/9 ]

[ 0  0    1 | 4 ]

Now the matrix is in reduced row-echelon form.

We can read the solution directly from the last column:

x = 8 - (1 + 8) = -1

y = 82/9 - (16/9)(4) = 2

z = 4

Therefore, the solution to the system of equations is (-1, 2, 4).

To learn more about the system of equations;

brainly.com/question/13729904

#SPJ7

Boxes A and B contained 112 pencils. When 1/5 of the pencils in box A were transferred to box B, both boxes combined the same number of pencils. How many more pencils were there in box A than in Box B at first?

Answers

Answer:

The number of pencils in the box A is 70

The number of pencils in the box B is 42

Step-by-step explanation:

The number of pencils in the box A is a

The number of pencils in the box B is b

We have:

a + b = 112

a - (1/5)a = b + (1/5)a

or

a + b = 112

a - (2/5)a = b

or

a + b = 112

(3/5)a = b

or

a + (3/5)a = 112

or

(8/5)a = 112

or

a = 112*(5/8)

or

a = 70

=> b = 112 - 70 = 42

Hope this helps!

A 70
B 42
Hope this helps

Use slopes and y-intercepts to determine if the lines y=5x+5 and 5x−y=−5 are parallel.

Answers

Answer: They are not parallel, they are coincident

Step-by-step explanation:

If two lines have the same slope but a different y-intercept, the lines are parallel. If two lines have the same slope and the same y-intercept, the lines are coincident.

We can rewrite 5x−y=−5 adding -5x to both sides and multiplying by -1:

5x - y =-5

5x - y -5x = -5 - 5x (adding -5x to both sides)

-y = -5 - 5x

Multiplying by -1

y = 5x + 5

Both equations look the same so they are coincident. They have the same intercept y=5 and the same slope m=5.

A company has a policy of retiring company cars; this policy looks at number of miles driven, purpose of trips, style of car and other features. The distribution of the number of months in service for the fleet of cars is bell-shaped and has a mean of 64 months and a standard deviation of 7 months. Using the empirical (68-95-99.7) rule, what is the approximate percentage of cars that remain in service between 43 and 50 months?

Answers

Answer:

[tex] z =\frac{50-64}{7}= -2[/tex]

[tex] z =\frac{43-64}{7}= -3[/tex]

We know that within two deviations from the mean we have 95% of the data from the empirical rule so then below 2 deviation from the mean we have (100-95)/2 % =2.5%. And within 3 deviations from the mean we have 99.7% of the data so then below 3 deviations from the mean we have (100-99.7)/2% =0.15%

And then the final answer for this case would be:

[tex] 2.5 -0.15 = 2.35\%[/tex]

Step-by-step explanation:

For this case we have the following parameters from the variable number of motnhs in service for the fleet of cars

[tex] \mu = 64, \sigma =7[/tex]

For this case we want to find the percentage of values between :

[tex] P(43< X< 50)[/tex]

And we can use the z score formula given by:

[tex] z = \frac{X-\mu}{\sigma}[/tex]

In order to calculate how many deviation we are within from the mean. Using this formula for the limits we got:

[tex] z =\frac{50-64}{7}= -2[/tex]

[tex] z =\frac{43-64}{7}= -3[/tex]

We know that within two deviations from the mean we have 95% of the data from the empirical rule so then below 2 deviation from the mean we have (100-95)/2 % =2.5%. And within 3 deviations from the mean we have 99.7% of the data so then below 3 deviations from the mean we have (100-99.7)/2% =0.15%

And then the final answer for this case would be:

[tex] 2.5 -0.15 = 2.35\%[/tex]

Select all statements below that are true about the binomial distribution shown on the right. The bar for any number k represents the probability of getting k successes in 5 flips. The number of successes, k, can range from 0 (no success) to 5 (all successes). Each coin flip is independent; it is not affected by any other coin flip. For 5 coin flips, P(2 heads) = P(3 heads). The sum of the probabilities shown in the binomial distribution is p.

Answers

Answer:

Step-by-step explanation:

Hello!

Distribution in attachment.

The variable of interest is:

X: Number of successes after flipping a coin 5 times.

If you check the binomial criteria:

The number of trials is fixed: n=5

There are only two possible outcomes "success" or "failure"

Each flip of the coin is independent of the others.

The probability of success in the same from one trial to another, in this case, if we consider the coin to be balanced, the probability of success is p=0.5

The histogram shows the probability of obtaining X number of success in 5ve flips of a coin (y-axis) vs the number of successes counted each (x-axis)

Statements:

1) The bar for any number k represents the probability of getting k successes in 5 flips. Correct. The histogram shows the probability of obtaining X number of success in 5ve flips of a coin (y-axis) vs the number of successes counted each (x-axis). Each bar represents the probability of success for each possible value.

2) The number of successes, k, can range from 0 (no success) to 5 (all successes). Correct. The variable count the number of successes after flipping a coin 5 times. It can happen that you flip it and all the flips turn to be failures (X=0), that you flip it 5 times and only one turns out to be a success and the other 4 are failures (X=1), and so on until you flip it 5 times and all flips are successes (X=5)

3) Each coin flip is independent; it is not affected by any other coin flip. Correct, if not, this variable wouldn't have a binomial distribution as specified in the text.

4) For 5 coin flips, P(2 heads) = P(3 heads). Correct

Looking at the histogram, the bars for "2 successes" and "3 successes" have the same height, a little above 0.3, this means that both values have the same probability of occurrence.

5) The sum of the probabilities shown in the binomial distribution is p.

Incorrect.

For the binomial distribution "p" represents the probability of success for each trial, in this case, flipping the coin once.

For this distribution, as well as for other probability distribution, the sum of all probabilities is always 1, if not, then it is not a probability distribution.

I hope this helps!

Answer:

Lets make this easier for you guys. The correct answers are 1 through 4.

The bar for any number k represents the probability of getting k successes in 5 flips.

The number of successes, k, can range from 0 (no success) to 5 (all successes).

Each coin flip is independent; it is not affected by any other coin flip.

For 5 coin flips, P(2 heads) = P(3 heads

Step-by-step explanation:

Correct on edge :))

A nurse’s aide earns $375 per week for 50 weeks of the year. What are her total earnings for the year?

Answers

Answer:

$18,750

Step-by-step explanation:

Take the earnings per week times the number of weeks

50 * 375

18,750

Answer:

$18,750

Step-by-step explanation:

If we want to find the nurse's aide's earnings for the year, we have to multiply her weekly salary by the number of weeks she worked.

weekly salary * number of weeks

She earns $375 per week and she worked for 50 weeks.

$375*50 weeks

375*50

Multiply the 2 numbers

18,750

Her total earnings for the year are $18,750

In a certain area an average of 13 new swarms of honeybees are seen each spring. If the number of swarms stays constant each year, what is the probability of observing between 9 and 15 (inclusive) swarms?

Answers

Answer:

The probability of observing between 9 and 15 (inclusive) swarms is 0.6639.

Step-by-step explanation:

The random variable X can be defined as the number of swarms of honeybees seen each spring.

The average value of the random variable X is, λ = 13.

A random variable representing the occurrence of events in a fixed interval of time is known as Poisson random variables.

For example, the number of customers visiting the bank in an hour or the number of typographical error is a book every 10 pages.

So, the random variable X follows a Poisson distribution with parameter λ = 13.

The probability mass function of X is as follows:

[tex]P(X=x)=\frac{e^{-\lambda}\ \lambda^{x}}{x!}; x=0,1,2,3...[/tex]

Compute the  the probability of observing between 9 and 15 (inclusive) swarms as follows:

P (9 ≤ X ≤ 15) = P (X = 9) + P (X = 10) + P (X = 11) + ... + P (X = 15)

                      [tex]=\sum\limits^{15}_{x=9}{\frac{e^{-\lambda}\ \lambda^{x}}{x!}}\\\\=0.06605+0.08587+0.10148+0.10994\\+0.10994+0.10209+0.08848\\\\=0.66385\\\\\approx 0.6639[/tex]

Thus, the probability of observing between 9 and 15 (inclusive) swarms is 0.6639.

Suppose that x has a binomial distribution with n = 201 and p = 0.45. (Round np and n(1-p) answers to 2 decimal places. Round your answers to 4 decimal places. Round z values to 2 decimal places. Round the intermediate value (σ) to 4 decimal places.) (a) Show that the normal approximation to the binomial can appropriately be used to calculate probabilities about x.

Answers

Answer:

a) It can be used because np and n(1-p) are both greater than 5.

Step-by-step explanation:

Binomial distribution and approximation to the normal:

The binomial distribution has two parameters:

n, which is the number of trials.

p, which is the probability of a success on a single trial.

If np and n(1-p) are both greater than 5, the normal approximation to the binomial can appropriately be used.

In this question:

[tex]n = 201, p = 0.45[/tex]

So, lets verify the conditions:

np = 201*0.45 = 90.45 > 5

n(1-p) = 201*(1-0.45) = 201*0.55 = 110.55 > 5

Since both np and n(1-p) are greater than 5, the approximation can be used.

The ice cream shop has 7 types of toppings available, and you decide to add 4 toppings to your bowl of 5 scoops of ice cream. How many combinations of 5 scoops of ice cream and 4 toppings are possible

Answers

Answer: 35

Step-by-step explanation:

The number of combinations of r things selected out of n things is given by

[tex]^nC_r= \dfrac{n!}{r!(n-r)!}[/tex]

Given , the total number of types of toppings available = 7

The number of toppings needed to be selected = 4

Then, the number of ways to do this would be

[tex]^7C_4=\dfrac{7!}{4!(7-4)!}\\\\=\dfrac{7\times6\times5\times4!}{4!3!}\\\\=\dfrac{7\times5}{1}=35[/tex]

Hence, the number of combinations of 5 scoops of ice cream and 4 toppings are possible = 35.

what do you think 40×40 is
And tell how you got your answer​

Answers

Answer:

1600

please see the attached picture for full solution

Hope it helps...

Solve the equation 4c=3
c =

Answers

Answer:−2x=−8

4sin2(x)−1=0

2x+3=3

Step-by-step explanation:

Can someone plz help me solved this problem I need help plz help me! Will mark you as brainiest!

Answers

Answer:

260

20

Step-by-step explanation:

speed of plane= p

speed of wind =w

(p+w)*3=840 p+w=840/3=280

and

(p-w)*3.5=840p-w= 840/3.5= 240

added up the 2 equations we get:

2p= 280+240p=260 mph

then

w= 20 mph

Please answer this correctly

Answers

Answer:

B) Mia Hamm helped her soccer team at the University of North Carolina at Chapel Hill win four NCAA titels.

Step-by-step explanation:

The first option is an opinion, not a fact.

Given the functions f(x) = 6x + 11 and g(x) = x^2 + 6, which of the following functions represents f[g(x)] correctly?

Answers

Answer:

Solve   -x2+11xandg+6x-6  = 0

Step-by-step explanation:

Suppose 221 subjects are treated with a drug that is used to treat pain and 51 of them developed nausea. Use a 0.10 significance level to test the claim that more than 20​% of users develop nausea.

Answers

Answer:

[tex]z=\frac{0.231 -0.2}{\sqrt{\frac{0.2(1-0.2)}{221}}}=1.152[/tex]  

The p avlue for this case is given by:

[tex]p_v =P(z>1.152)=0.125[/tex]  

The p value for this case is higher than the significance level so then we have enough evidence to FAIL to reject the null hypothesis and we can't conclude that the true proportion is higher than 0.2 or 20%

Step-by-step explanation:

Information provided

n=221 represent the random sample taken

X=51 represent the people with nausea

[tex]\hat p=\frac{51}{221}=0.231[/tex] estimated proportion of people with nausea

[tex]p_o=0.21[/tex] is the value to test

[tex]\alpha=0.1[/tex] represent the significance level

z would represent the statistic

[tex]p_v[/tex] represent the p value

Hypothesis to verify

We want to check if the true population is higher than 0.20, the system of hypothesis are.:  

Null hypothesis:[tex]p \leq 0.2[/tex]  

Alternative hypothesis:[tex]p > 0.2[/tex]  

The statistic is given:

[tex]z=\frac{\hat p -p_o}{\sqrt{\frac{p_o (1-p_o)}{n}}}[/tex] (1)  

Replacing the info given we got:

[tex]z=\frac{0.231 -0.2}{\sqrt{\frac{0.2(1-0.2)}{221}}}=1.152[/tex]  

The p avlue for this case is given by:

[tex]p_v =P(z>1.152)=0.125[/tex]  

The p value for this case is higher than the significance level so then we have enough evidence to FAIL to reject the null hypothesis and we can't conclude that the true proportion is higher than 0.2 or 20%

Budget
8.) If Peter Gower paid $650 for rent
monthly for an entirely year, how
much should he budget for rent
each month?
Answer:
I
(a) $108.33
(b) $54.17
(c) $7.800
(d) $650

Answers

My guess would be A ! Hope this helps

A cereal box is an example of a____ a0___ a1.

Answers

Answer:

rectangular prism

Step-by-step explanation:

A rectangular prism has 6 sides that are rectangles.

Which statement accurately describes chemical rocks?

Answers

Answer:

Chemical rocks form when minerals dissolve in a solution and crystalize.

Answer:

Chemical rocks don't form from solidification from a melt

Step-by-step explanation:

Salid bought 30 feet of window trim at a hardware store. The trim cost $1.75 per foot including sales tax. If Salid paid with a $100.00 bill, how much change should he have received?

Answers

Answer:

47.50

Step-by-step explanation:

According to the question above Salid bought 30 feet of window at a hardware trim store

The trim cost of each window is $1.75 per foot with an inclusion of sales tax added to this amount

= $1.75×30

= 52.5

Since Salid paid for the trim service with a cash of $100.00, his change is calculated as follows

=$100-52.5

= $47.50

Hence Salid change is $47.50

How many strings can be formed by ordering the letters MISSISSIPPI which


contain the substring of MISS?

Answers

Answer:

1680 is the answer.

Step-by-step explanation:

Here, we have 11 letters in the word MISSISSIPPI.

Repetition of letters:

M - 1 time

I - 4 times

S - 4 times

P - 2 times

As per question statement, we need a substring MISS in the resultant strings.

So, we need to treat MISS as one unit so that MISS always comes together in all the strings.

The resultant strings will look like:

xxxxMISSxxx

xxMISSxxxxx

and so on.

After we treat MISS as one unit, total letters = 8

Repetition of letters:

MISS - 1 time

I - 3 times

S - 2 times

P - 2 times

The formula for combination of letters with total of n letters:

[tex]\dfrac{n!}{p!q!r!}[/tex]

where p, q and r are the number of times other letters are getting repeated.

p = 3

q = 2

r = 2

So, required number of strings that contain MISS as substring:

[tex]\dfrac{8!}{3!2!2!}\\\Rightarrow \dfrac{40320}{6\times 2 \times 2}\\\Rightarrow 1680[/tex]

So, 1680 is the answer.

14d+21 and 7 (2d+3) choose yes or no to see if the expression is equivalent

Answers

Answer:

Yes- these answers are equivalent.

Step-by-step explanation:

In the expression 7(2d+3), the 7 is outside the parentheses, meaning everything inside the parentheses is multiplied by 7.

2d X 7 = 14 d

3 X 7 = 21

7(2d+3) = 14d + 21

Yes, the expressions are equivalent.

What is expression ?

By combining numbers, variables, functions in mathematics we get an expression.

Example : 4p+2, 3x-4y etc

What is the required result ?

Given expressions are 14d+21 and 7(2d+3)

Here, simplifying the expression 7(2d+3) we get,

7(2d+3) = (7×2d)+(7×3) = 14d+21

Therefore, the given two expressions are equal to each other, i.e same.

Learn more about expression here :

https://brainly.com/question/723406

#SPJ2

In performing a chi-square goodness-of-fit test with multinomial probabilities, the ___________ the difference between observed and expected frequencies, the higher the probability of concluding that the probabilities specified in the null hypothesis are correct.

Answers

Answer:

Step-by-step explanation:

The smaller/closer the difference between observed and expected frequencies, the higher the probability of concluding that the probabilities specified in the null hypothesis are correct concluding that the data fits that particular distribution given.

A recent gasoline survey said that the national average price of gasoline was $1.298 a gallon. It was felt that gasoline price in Texas was significantly lower than the national average. A study of 37 different suburbs in Dallas, Texas, found the average price of gasoline to be $1.192 a gallon with a standard deviation of $0.0436. What is the alternative hypothesis

Answers

Answer:

alternative hypothesis : H₁ :

Recent  Gasoline surveys felt that gasoline price in Texas was significantly lower than the national average

Alternative Hypothesis : H₁: μ < $1.298 a gallon

Step-by-step explanation:

Explanation:-

Given A recent gasoline survey said that the national average price of gasoline was $1.298 a gallon

Population average μ= $1.298 a gallon

sample size 'n' = 37

Sample mean (x⁻) = $1.192 a gallon

Sample standard deviation 'S' = $0.0436

Null hypothesis :H₀ : μ =  $1.298 a gallon

Alternative Hypothesis : μ < $1.298 a gallon

Degrees of freedom : ν = n-1= 37-1=36

t₀.₀₂₅ = 1.688

Test statistic

                       [tex]t = \frac{x^{-}-mean }{\frac{S}{\sqrt{n} } }[/tex]

                       [tex]t = \frac{1.192-1.298 }{\frac{0.0436}{\sqrt{37} } }[/tex]

                     t =  -14.8044

|t| = |-14.8044| > 1.688

Null hypothesis is rejected

Alternative hypothesis is accepted

Recent  Gasoline surveys felt that gasoline price in Texas was significantly lower than the national average

I can’t figure this out it’s difficult for can anyone help me Plz

Answers

Answer:

the correct option is D

Step-by-step explanation:

JK IS longer than JL

The answer is C. Hope this helps

A square matrix AA is called half-magic if the sum of the numbers in each row and column is the same. The common sum in each row and column is denoted by s(A)s(A) and is called the magic sum of the matrix AA. Let VV be the vector space of 2×22×2 half-magic squares.
A) Find an ordered basis BB for VV.
B) Find the coordinate vector [M]_B of M [-2 -7, -7 -2]

Answers

Answer:

A)  [tex]B = \{\left[\begin{array}{ccc}1&0\\0&1 \end{array}\right], \left[\begin{array}{ccc}0&1\\1&0 \end{array}\right] \}[/tex]

B) [tex]M_{B} = \left[\begin{array}{ccc}-2\\-7\end{array}\right][/tex]

Step-by-step explanation:

Let [tex]A = \left[\begin{array}{ccc}a&b\\c&d \end{array}\right][/tex] where a, b, c and d are real numbers

Since A is said to be a half magic square matrix, a = d, b = c.

The matrix A therefore becomes  [tex]A = \left[\begin{array}{ccc}a&b\\b&a \end{array}\right][/tex] where [tex]a,b \epsilon R[/tex]

A can therefore be manipulated as:

[tex]A = a \left[\begin{array}{ccc}1&0\\0&1 \end{array}\right] + b \left[\begin{array}{ccc}0&1\\1&0 \end{array}\right][/tex]

The matrices [tex]\left[\begin{array}{ccc}1&0\\0&1 \end{array}\right][/tex] and [tex]\left[\begin{array}{ccc}0&1\\1&0 \end{array}\right][/tex] are apparently linearly independent and therefore form a basis B for V

[tex]B = \{\left[\begin{array}{ccc}1&0\\0&1 \end{array}\right], \left[\begin{array}{ccc}0&1\\1&0 \end{array}\right] \}[/tex]

B) Find the coordinate vector [M]_B of M [-2 -7, -7 -2]

 [tex]M = \left[\begin{array}{ccc}-2&-7\\-7&-2 \end{array}\right][/tex]

[tex]M[/tex] can be written in the form [tex]M = a\left[\begin{array}{ccc}1&0\\0&1 \end{array}\right] + b\left[\begin{array}{ccc}0&1\\1&0 \end{array}\right][/tex]

[tex]M = \left[\begin{array}{ccc}-2&-7\\-7&-2 \end{array}\right] = -2\left[\begin{array}{ccc}1&0\\0&1 \end{array}\right] -7\left[\begin{array}{ccc}0&1\\1&0 \end{array}\right][/tex]

The coordinate vector is therefore, [tex]M_{B} = \left[\begin{array}{ccc}-2\\-7\end{array}\right][/tex]

Other Questions
Which of the following provides the best summary of the process of naturalselection?A. Individuals always change in response to their environment.B. Genes are passed from parent to offspring.c. Mutations always increase an individual's fitness.D. Individuals that are well adapted to their environment survive. Which succession occurs in a community that was never colonized before? A. secondary B. artificial C. temporary D.primary If ladders are difficult to use for high floors, what are some other options? What is thelevel of safety of each of your options? Explain using mathematical facts. How do details from the excerpts support the purpose ofadvocating for children from Central America?Both include information that ask people to help childrenfrom Central America.Both offer solutions that show how victims from othercountries have been helped.The excerpt from Enrique's Journey uses quotations toshow that it is better for these children to be sent back,and "Children of the Drug Wars" uses examplesdescribing how other countries have dealt with the issueThe excerpt from Enrique's Journey tells a story aboutwhat happened to one victim, and "Children of the DrugWars" uses words that create an emotional response topersuade readers to take action. A house on the market was valued at $295,000. After several years, the value increased by 8%. By how much did the houses value increase in dollars? What is the current value of the house? a man climbs on to a wall that is 4.5m high and gains 2450j 0f potential energy.what is the mass of the man? Oh. Jons was silent for a minute. Oh, I see what you mean. It wouldnt matter for a new childs toy. But later it does matter, doesnt it? We dont dare to let people make choices of their own. How does this paragraph relate to the theme of the story? Use two details from the story to support your response. Which of the following is a statement of inductive reasoning? answers A) I like this movie; all movies are entertaining. B) This memory is good; this memory is a function of the brain; all functions of the brain are good. C) Hope is good; hope saved Elie Wiesel; Elie Wiesel is good. D) Cinderella is a princess; princesses are good; Cinderella is good. 80,+84, +82, +79, +72, +97, +89, +97, +88, +79, +86, +90, +87 = 2. Convert 6.578 x 1023 atoms of magnesium to moles.6.578 x 1023 atoms xmolatoms On the moon, the acceleration of gravity is 1.6 m/s2. If an object has amass on Earth of 5.1 kg, what is its mass on the moon? Question 9 of 103 PointsSuppose a merchant in a moneyless economy traded his goods for a largeamount of purple fabric. He intended to use the fabric to trade for othergoods in another location. While he traveled to the other marketplace,however, a trade ship arrived with a large shipment of purple fabric. As aresult, the merchant was only able to trade his fabric for about half of what hehad expected it to be worth in return. This situation illustrates the need forwhat aspect of the purpose of money?A. Preservation of valueB. Unit of measureC.Ease of exchangeD.Government endorsement I NEED HELP PLEASE!! What is the best way to check the reliability and validity of a source Which of the following describes slavery in the northern colonies in the 18th century The equation 1/x=0 has no solution. True or False 60f=86 and I need to divide 60 from both sides but I dont know how Read the paragraph. The schoolwide food drive is an important program that helps the community and instills a sense of responsibility in students. Last year, students collected over 600 canned goods and gave them to the Neighborhood House food pantry, which serves some of the neediest families in our community. Furthermore, many students said that helping with the drive was the first volunteer program they had ever participated in. Finally, without the student councils support, the food drive will not be officially promoted within school and will likely languish. Therefore, it is imperative that the student council continue to encourage students to volunteer for this important program. What answer makes the best additional evidence to support the authors reason? A poll of one hundred students found that after participating in the food drive, seventy-three participants found the experience so rewarding that they sought out new volunteer opportunities. My friend Alvin participated in the food drive and said not only was it the best part of the school year, but he looks forward to helping out with the food drive next year. We can definitely beat our previous goal of six hundred cans by both encouraging additional students to join the program and offering prizes for students who collect the most cans. The student council needs to take a leadership role and promote the food drive so that all students, regardless of their previous volunteer experience, become active participants. Platelets are responsible for _____. carrying glucose to cells transfusing of blood carrying oxygen to cells clotting of blood please help me! Explain what Martin Luther King and Rosa Parks accomplished with the Montgomery Bus Boycott? Do you think it was difficult for them to get people to not ride the bus in Montgomery? PLEASE HELP ME!MJ WORLD HISTORY!